Proof related to derivative and Big O notation

  • #1
songoku
2,296
325
Homework Statement
Please see below
Relevant Equations
Limit

Derivative
1700879114137.png


My attempt:
Since ##\frac{f(a+h)-f(a-h)}{2h}-f'(a)=O(h^2)## as ##h \to 0##, then:

$$\lim_{h \to 0} \frac{\frac{f(a+h)-f(a-h)}{2h}-f'(a)}{h^2} < \infty$$

So

$$\lim_{h \to 0} \frac{\frac{f(a+h)-f(a-h)}{2h}-f'(a)}{h^2} = \lim_{h \to 0} \frac{f'(a)-f'(a)}{h^2}=0 < \infty$$

Because the value of the limit is zero, it is true that ##\frac{f(a+h)-f(a-h)}{2h}-f'(a)=O(h^2)## as ##h \to 0##

Is this even correct?

Thanks
 
Physics news on Phys.org
  • #2
This step:
$$\lim_{h \to 0} \frac{\frac{f(a+h)-f(a-h)}{2h}-f'(a)}{h^2} = \lim_{h \to 0} \frac{f'(a)-f'(a)}{h^2}$$
is incorrect.
 
  • Like
Likes songoku
  • #3
Hill said:
This step:
$$\lim_{h \to 0} \frac{\frac{f(a+h)-f(a-h)}{2h}-f'(a)}{h^2} = \lim_{h \to 0} \frac{f'(a)-f'(a)}{h^2}$$
is incorrect.
$$\lim_{h \to 0} \frac{\frac{f(a+h)-f(a-h)}{2h}-f'(a)}{h^2} $$
$$=\frac{\lim_{h \to 0} \frac{f(a+h)-f(a-h)}{2h}-\lim_{h \to 0} f'(a)}{\lim_{h \to 0} h^2}$$
$$= \frac{f'(a)-f'(a)}{\lim_{h \to 0} h^2}$$
$$=\frac{0}{\lim_{h \to 0} h^2}$$
$$=0$$

I can't do that? Thanks
 
  • #4
songoku said:
Homework Statement: Please see below
Relevant Equations: Limit

Derivative

View attachment 336113

My attempt:
Since ##\frac{f(a+h)-f(a-h)}{2h}-f'(a)=O(h^2)## as ##h \to 0##, then:
I'm confused. Isn't this what you are supposed to prove?
songoku said:
$$\lim_{h \to 0} \frac{\frac{f(a+h)-f(a-h)}{2h}-f'(a)}{h^2} < \infty$$

So

$$\lim_{h \to 0} \frac{\frac{f(a+h)-f(a-h)}{2h}-f'(a)}{h^2} = \lim_{h \to 0} \frac{f'(a)-f'(a)}{h^2}=0 < \infty$$

Because the value of the limit is zero, it is true that ##\frac{f(a+h)-f(a-h)}{2h}-f'(a)=O(h^2)## as ##h \to 0##
I don't see where you have used the fact that ##f## has continuous derivatives up to order 3 in an open interval containing ##a##
songoku said:
Is this even correct?
I think not.
 
  • Like
Likes songoku
  • #5
FactChecker said:
I'm confused. Isn't this what you are supposed to prove?

I don't see where you have used the fact that ##f## has continuous derivatives up to order 3 in an open interval containing ##a##
Yeah, actually that would be my next question after discussing my really bad approach. How to use the third and/or second derivative to prove the question? I don't see any higher-order derivative than 1 in the question, that's why I started from the limit.

Thanks
 
  • #6
songoku said:
How to use the third and/or second derivative to prove the question? I don't see any higher-order derivative than 1 in the question, that's why I started from the limit.
Have you learned about Taylor series? Are you allowed to use them for this problem?
 
  • Like
Likes songoku
  • #7
songoku said:
$$\lim_{h \to 0} \frac{\frac{f(a+h)-f(a-h)}{2h}-f'(a)}{h^2} $$
$$=\frac{\lim_{h \to 0} \frac{f(a+h)-f(a-h)}{2h}-\lim_{h \to 0} f'(a)}{\lim_{h \to 0} h^2}$$
$$= \frac{f'(a)-f'(a)}{\lim_{h \to 0} h^2}$$
$$=\frac{0}{\lim_{h \to 0} h^2}$$
$$=0$$

I can't do that? Thanks
No, on your last step above, instead of $$=\frac{0}{\lim_{h \to 0} h^2}=0$$ you'd get $$=\frac{0}{\lim_{h \to 0} h^2}=\frac 0 0$$
 
  • Like
Likes songoku
  • #8
renormalize said:
Have you learned about Taylor series? Are you allowed to use them for this problem?
The problem may include functions for which the Taylor series does not exist. This is pure mathematics!
 
  • #9
songoku said:
Yeah, actually that would be my next question after discussing my really bad approach. How to use the third and/or second derivative to prove the question? I don't see any higher-order derivative than 1 in the question, that's why I started from the limit.

Thanks
This looks tricky. It's not clear to me why you need derivatives up to order three. You'd think that ##f'## being continuous would be sufficient. If not, then the solution must get quite involved. Those are my initial thoughts.

Possibly start with the continuity of ##f'''##? And see what you get starting from there?
 
  • Like
Likes songoku
  • #10
The existence of three derivatives allows to apply L'Hôpital's rule three times. This is all we need.
 
  • Like
Likes songoku and PeroK
  • #11
My understanding is that you are asked to show the equivalence of your definition of a derivative (I can only guess that this is the usual definition as a limit) and Weierstraß's formula
$$
f(a+h)=f(a)+ J_a(h) + r(h)
$$
where ##J_a(h)=f'(a)\cdot h## is the Jacobi matrix of ##f## that is the derivarive at ##x=a.## ##r(h)## is the remainder that tends faster to zero than ##h,## i.e. ##\lim_{h \to 0}\dfrac{r(h)}{h}=0.##

You do not need a Taylor series for that. It is just the linear approximation of ##f## by its derivative ##f'## at ##x=a##. All higher-order terms of the Taylor series are in ##O(h^2)## or ##r(h).## You only need the separation of the linear term and all the rest. Taylor would be an overkill since there would be nothing left to prove. Use the definitions of a derivative and a limit, that's all.
 
  • Like
Likes songoku
  • #12
songoku said:
Yeah, actually that would be my next question after discussing my really bad approach. How to use the third and/or second derivative to prove the question? I don't see any higher-order derivative than 1 in the question, that's why I started from the limit.

Thanks
What is the context of this question? I was wondering whether this is in connection with the formal development of Taylor series? In any case, I've found Taylor's Theorem:

https://en.wikipedia.org/wiki/Taylor's_theorem#Taylor's_theorem_in_one_real_variable
 
  • Like
Likes songoku
  • #13
renormalize said:
Have you learned about Taylor series? Are you allowed to use them for this problem?
Yes I have and I am allowed to use them for this problem. It is just I don't know how to use them :redface:

Hill said:
No, on your last step above, instead of $$=\frac{0}{\lim_{h \to 0} h^2}=0$$ you'd get $$=\frac{0}{\lim_{h \to 0} h^2}=\frac 0 0$$
Oh ok, I see it now.

PeroK said:
What is the context of this question? I was wondering whether this is in connection with the formal development of Taylor series? In any case, I've found Taylor's Theorem:

https://en.wikipedia.org/wiki/Taylor's_theorem#Taylor's_theorem_in_one_real_variable
This is an exercise for practice for test as part of Calculus course, covering topics starting from function (domain, range, etc) up until Taylor (including limit, derivative, integration, ode).

fresh_42 said:
My understanding is that you are asked to show the equivalence of your definition of a derivative (I can only guess that this is the usual definition as a limit) and Weierstraß's formula
$$
f(a+h)=f(a)+ J_a(h) + r(h)
$$
where ##J_a(h)=f'(a)\cdot h## is the Jacobi matrix of ##f## that is the derivarive at ##x=a.## ##r(h)## is the remainder that tends faster to zero than ##h,## i.e. ##\lim_{h \to 0}\dfrac{r(h)}{h}=0.##

You do not need a Taylor series for that. It is just the linear approximation of ##f## by its derivative ##f'## at ##x=a##. All higher-order terms of the Taylor series are in ##O(h^2)## or ##r(h).## You only need the separation of the linear term and all the rest. Taylor would be an overkill since there would be nothing left to prove. Use the definitions of a derivative and a limit, that's all.
$$1) \lim_{h\to 0} \frac{f(a+h)-f(a)}{h}=f'(a)$$
$$2) \lim_{h\to 0} \frac{f'(a+h)-f'(a)}{h}=f"(a)$$
$$3) \lim_{h\to 0} \frac{f"(a+h)-f"(a)}{h}=f^{3} (a)$$
$$4) \lim_{h\to 0} \frac{f(a+h)-f(a-h)}{2h}=f'(a)$$
$$5) \lim_{h\to 0} \frac{f(a+h)-f(a-h)}{h^2}=f"(a)$$

Do you mean that is the starting point? Thanks
 
  • #14
songoku said:
$$1) \lim_{h\to 0} \frac{f(a+h)-f(a)}{h}=f'(a)$$
$$2) \lim_{h\to 0} \frac{f'(a+h)-f'(a)}{h}=f"(a)$$
$$3) \lim_{h\to 0} \frac{f"(a+h)-f"(a)}{h}=f^{3} (a)$$
$$4) \lim_{h\to 0} \frac{f(a+h)-f(a-h)}{2h}=f'(a)$$
$$5) \lim_{h\to 0} \frac{f(a+h)-f(a-h)}{h^2}=f"(a)$$

Do you mean that is the starting point? Thanks
Yes.

If we have Weierstraß ##f(a+h)=f(a) +f'(a)\cdot h +r(h)## with ##\lim_{h \to 0}\dfrac{r(h)}{h}=0## then ##f(a-h)=f(a) -f'(a)\cdot h +r(h)## and ##\dfrac{f(a+h)-f(a-h)}{2}=f'(a)\cdot h + r(h)## where ##r(h)## are possibly different remainders. However, ##\lim_{h \to 0}\dfrac{r(h)}{h}=0## holds for all of them. So writing the remainder as a power series yields
$$
\dfrac{r(h)}{h}=\dfrac{r_0}{h}=\dfrac{r_0}{h}+r_1+r_2h+O(h^2) \stackrel{h\to 0}{\longrightarrow }0
$$
and therefore ##r_0=r_1=0## and ##r(h)=r_2h^2+O(h^3)=O(h^2).##

The Weierstraß decomposition formula is equivalent to differentiability.
(e.g. https://de.wikipedia.org/wiki/Weierstraßsche_Zerlegungsformel#Beweis)
I haven't searched for an English version, but the formulas should do, or you use Chrome and let it translate the page to English (right-click), or you look on the internet for an English proof, or you prove it as an exercise, which I assumed this exercise is all about.
 
Last edited:
  • Like
Likes songoku
  • #15
songoku said:
This is an exercise for practice for test as part of Calculus course, covering topics starting from function (domain, range, etc) up until Taylor (including limit, derivative, integration, ode).
Use L'Hopital's rule, as suggested above.
 
  • Like
Likes songoku and Hill
  • #16
fresh_42 said:
$$
\dfrac{r(h)}{h}=\dfrac{r_0}{h}=\dfrac{r_0}{h}+r_1+r_2h+O(h^2) \stackrel{h\to 0}{\longrightarrow }0
$$
and therefore ##r_0=r_1=0## and ##r(h)=r_2h^2+O(h^3)=O(h^2).##
Sorry I don't understand this part. Why ##\dfrac{r(h)}{h}=\dfrac{r_0}{h}?## Then shouldn't it be ##r(h)=r_0##?

And also why ##r_0=r_1=0##?

PeroK said:
Use L'Hopital's rule, as suggested above.
$$\lim_{h\to 0} \frac{\frac{f(a+h)-f(a-h)}{2h}-f'(a)}{h^2}$$
$$=\lim_{h\to 0}\frac{f(a+h)-f(a-h)-f'(a).2h}{2h^3}$$
$$=\lim_{h\to 0} \frac{f'(a+h)+f'(a-h)-2f'(a)}{6h^2}$$
$$=\lim_{h\to 0} \frac{f"(a+h)-f"(a-h)}{12h}$$
$$=\lim_{h\to 0} \frac{f^3 (a+h)+f^3 (a+h)}{12}$$
$$=\frac{f^3 (a)}{6}$$

Is this what you mean?

Thanks
 
  • #17
That's exactly what @Hill and I meant!
 
  • Like
Likes songoku and Hill
  • #18
songoku said:
Sorry I don't understand this part. Why ##\dfrac{r(h)}{h}=\dfrac{r_0}{h}?## Then shouldn't it be ##r(h)=r_0##?

And also why ##r_0=r_1=0##?$$\lim_{h\to 0} \frac{\frac{f(a+h)-f(a-h)}{2h}-f'(a)}{h^2}$$
$$=\lim_{h\to 0}\frac{f(a+h)-f(a-h)-f'(a).2h}{2h^3}$$
$$=\lim_{h\to 0} \frac{f'(a+h)+f'(a-h)-2f'(a)}{6h^2}$$
$$=\lim_{h\to 0} \frac{f"(a+h)-f"(a-h)}{12h}$$
$$=\lim_{h\to 0} \frac{f^3 (a+h)+f^3 (a+h)}{12}$$
$$=\frac{f^3 (a)}{6}$$

Is this what you mean?

Thanks
Except a small typo in the line before the last.

P.S. This is where the continuity of the third derivative is used.
 
  • Like
Likes songoku and PeroK
  • #19
Say ##r(h)=\sum_{k=0}^\infty r_kh^k.## Then
$$
0=\lim_{h \to 0}\dfrac{r(h)}{h}=\lim_{h \to 0}\sum_{k=0}^\infty r_kh^{k-1}\stackrel{h\to 0}{=}\lim_{h \to 0}\sum_{k=0}^1 r_kh^{k-1}=\lim_{h \to 0}\left(\dfrac{r_0}{h}+r_1\right)
$$
which can only be if ##r_0=r_1=0.## Thus ##r(h)=\sum_{k=2}^\infty r_kh^k=O(h^2).##
 
Last edited:
  • Like
Likes songoku
  • #20
Thank you very much for all the help and explanation Hill, FactChecker, renormalize, PeroK, fresh_42
 
  • Like
Likes Hill and fresh_42

Similar threads

  • Calculus and Beyond Homework Help
Replies
12
Views
323
  • Calculus and Beyond Homework Help
Replies
20
Views
1K
  • Calculus and Beyond Homework Help
Replies
2
Views
916
  • Calculus and Beyond Homework Help
Replies
7
Views
1K
  • Calculus and Beyond Homework Help
Replies
2
Views
226
  • Calculus and Beyond Homework Help
Replies
12
Views
1K
  • Calculus and Beyond Homework Help
Replies
8
Views
510
  • Calculus and Beyond Homework Help
Replies
6
Views
1K
  • Calculus and Beyond Homework Help
Replies
4
Views
861
  • Calculus and Beyond Homework Help
Replies
9
Views
4K
Back
Top